www.vorkurse.de
Ein Projekt von vorhilfe.de
Die Online-Kurse der Vorhilfe

E-Learning leicht gemacht.
Hallo Gast!einloggen | registrieren ]
Startseite · Mitglieder · Teams · Forum · Wissen · Kurse · Impressum
Forenbaum
^ Forenbaum
Status Mathe-Vorkurse
  Status Organisatorisches
  Status Schule
    Status Wiederholung Algebra
    Status Einführung Analysis
    Status Einführung Analytisc
    Status VK 21: Mathematik 6.
    Status VK 37: Kurvendiskussionen
    Status VK Abivorbereitungen
  Status Universität
    Status Lerngruppe LinAlg
    Status VK 13 Analysis I FH
    Status Algebra 2006
    Status VK 22: Algebra 2007
    Status GruMiHH 06
    Status VK 58: Algebra 1
    Status VK 59: Lineare Algebra
    Status VK 60: Analysis
    Status Wahrscheinlichkeitst

Gezeigt werden alle Foren bis zur Tiefe 2

Navigation
 Startseite...
 Neuerdings beta neu
 Forum...
 vorwissen...
 vorkurse...
 Werkzeuge...
 Nachhilfevermittlung beta...
 Online-Spiele beta
 Suchen
 Verein...
 Impressum
Das Projekt
Server und Internetanbindung werden durch Spenden finanziert.
Organisiert wird das Projekt von unserem Koordinatorenteam.
Hunderte Mitglieder helfen ehrenamtlich in unseren moderierten Foren.
Anbieter der Seite ist der gemeinnützige Verein "Vorhilfe.de e.V.".
Partnerseiten
Weitere Fächer:

Open Source FunktionenplotterFunkyPlot: Kostenloser und quelloffener Funktionenplotter für Linux und andere Betriebssysteme
Forum "Integralrechnung" - elliptisches Integral
elliptisches Integral < Integralrechnung < Analysis < Oberstufe < Schule < Mathe < Vorhilfe
Ansicht: [ geschachtelt ] | ^ Forum "Integralrechnung"  | ^^ Alle Foren  | ^ Forenbaum  | Materialien

elliptisches Integral: Frage (beantwortet)
Status: (Frage) beantwortet Status 
Datum: 16:41 Di 24.08.2010
Autor: Martinius

Aufgabe 1
Die Gleichung einer Ellipse lautet:  

[mm] $\bruch{x^2}{a^2}+\bruch{y^2}{b^2}=1$ [/mm]

Schreibe eine Formel für den Umfang einer Ellipse auf und forme das Integral in eine einfache Form um.

Das Integral heißt elliptisches Integral, weil es den Umfang einer Ellipse angibt. Die Integrandenfunktion ist nicht elementar integrierbar.

Aufgabe 2
Das Lösungsbuch sagt:

Umfang einer Ellipse:

Die Ellipse [mm] $\bruch{x^2}{a^2}+\bruch{y^2}{b^2}=1$ [/mm]

mit der numerischen Exzentrizität [mm] k=sin(\alpha)=\bruch{\sqrt{a^2-b^2}}{a} [/mm]

hat den Umfang

[mm] U=4a\int_{0}^{\pi/2}\sqrt{1-k^2sin^2(\varphi)}\,d \varphi=4aE\left(k;\bruch{\pi}{2}\right) [/mm]

Hallo,

mir ist etwas schleierhaft, wie man von

[mm] $\bruch{x^2}{a^2}+\bruch{y^2}{b^2}=1$ [/mm]

[mm] $y=\sqrt{b^2-\bruch{b^2}{a^2}*x^2}=\bruch{b}{a}*\sqrt{a^2-x^2}$ [/mm]

[mm] $y'=\bruch{b}{a}*\bruch{-x}{\sqrt{a^2-x^2}}$ [/mm]


[mm] $s=2*\int_{-a}^{a}\sqrt{1+(y'(x))^2}\,dx=4*\int_{0}^{a}\sqrt{1+\bruch{b^2}{a^2}*\bruch{x^2}{a^2-x^2}}\,dx=4*\bruch{b}{a}*\int_{0}^{a}\sqrt{\bruch{a^2}{b^2}+\bruch{x^2}{a^2-x^2}}\,dx$ [/mm]

auf

[mm] U=4a\int_{0}^{\pi/2}\sqrt{1-k^2sin^2(\varphi)}\,d \varphi=4aE\left(k;\bruch{\pi}{2}\right) [/mm]

kommt - was wohl die Legendre-Form eines unvollständigen elliptischen Integrals der 1. Art sein soll.

Vielen Dank für einen Hinweis.

LG, Martinius

        
Bezug
elliptisches Integral: Parameterdarstellung
Status: (Antwort) fertig Status 
Datum: 17:59 Di 24.08.2010
Autor: Al-Chwarizmi


> Die Gleichung einer Ellipse lautet:  
>
> [mm]\bruch{x^2}{a^2}+\bruch{y^2}{b^2}=1[/mm]
>  
> Schreibe eine Formel für den Umfang einer Ellipse auf und
> forme das Integral in eine einfache Form um.
>  
> Das Integral heißt elliptisches Integral, weil es den
> Umfang einer Ellipse angibt. Die Integrandenfunktion ist
> nicht elementar integrierbar.
>  Das Lösungsbuch sagt:
>  
> Umfang einer Ellipse:
>  
> Die Ellipse [mm]\bruch{x^2}{a^2}+\bruch{y^2}{b^2}=1[/mm]
>
> mit der numerischen Exzentrizität
> [mm]k=sin(\alpha)=\bruch{\sqrt{a^2-b^2}}{a}[/mm]
>  
> hat den Umfang
>
> [mm]U=4a\int_{0}^{\pi/2}\sqrt{1-k^2sin^2(\varphi)}\,d \varphi=4aE\left(k;\bruch{\pi}{2}\right)[/mm]
>  
> Hallo,
>  
> mir ist etwas schleierhaft, wie man von
>  
> [mm]\bruch{x^2}{a^2}+\bruch{y^2}{b^2}=1[/mm]
>  
> [mm]y=\sqrt{b^2-\bruch{b^2}{a^2}*x^2}=\bruch{b}{a}*\sqrt{a^2-x^2}[/mm]
>  
> [mm]y'=\bruch{b}{a}*\bruch{-x}{\sqrt{a^2-x^2}}[/mm]
>  
>
> [mm]s=2*\int_{-a}^{a}\sqrt{1+(y'(x))^2}\,dx=4*\int_{0}^{a}\sqrt{1+\bruch{b^2}{a^2}*\bruch{x^2}{a^2-x^2}}\,dx=4*\bruch{b}{a}*\int_{0}^{a}\sqrt{\bruch{a^2}{b^2}+\bruch{x^2}{a^2-x^2}}\,dx[/mm]
>  
> auf
>  
> [mm]U=4a\int_{0}^{\pi/2}\sqrt{1-k^2sin^2(\varphi)}\,d \varphi=4aE\left(k;\bruch{\pi}{2}\right)[/mm]
>  
> kommt - was wohl die Legendre-Form eines unvollständigen
> elliptischen Integrals der 1. Art sein soll.
>  
> Vielen Dank für einen Hinweis.
>  
> LG, Martinius


Hallo Martinius,

ich denke, dass deine Idee, vom Kurvenlängenintegral in recht-
winkligen Koordinaten auszugehen und dieses dann in eines über
Polarkoordinaten umzuwandeln, in der Durchführung schwierig
wird, wie du ja schon festgestellt hast.
Besser ist es, gleich mit der Parameterdarstellung der Ellipse zu
beginnen. Den Winkel nenne ich der Einfachheit des Schreibens
halber lieber t anstatt [mm] \varphi [/mm] . Die Gleichung der Ellipse ist dann:

      [mm] $\vektor{x\\y}(t)\ [/mm] =\ [mm] \vektor{a*cos(t)\\b*sin(t)}\qquad 0\le t\le 2*\pi [/mm] $

Wenn man die doppelte Axialsymmetrie der Ellipse nutzt, kann man
sich auf das Intervall  [mm] 0\le t\le \frac{\pi}{2} [/mm]  beschränken und das Ergebnis mit
4 multiplizieren. Jetzt brauchst du nur noch die richtige Formel
für die Bogenlänge in dieser Parameterdarstellung.


LG     Al-Chwarizmi

Bezug
                
Bezug
elliptisches Integral: Frage (beantwortet)
Status: (Frage) beantwortet Status 
Datum: 16:31 Mi 25.08.2010
Autor: Martinius

Hallo Al Chwarizmi,

besten Dank für die Antwort - dadurch bin ich der Lösung näher gekommen; ich habe aber noch folgendes Problem:

Die Parametergleichung für die Ellipse ist ja:

$ [mm] \vektor{x(t)\\y(t)}\ [/mm] =\ [mm] \vektor{a\cdot{}cos(t)\\b\cdot{}sin(t)}\qquad 0\le t\le 2\cdot{}\pi [/mm] $


Und ihre erste Ableitung:

$ [mm] \vektor{ \dot x(t)\\\dot y(t)}\ [/mm] =\ [mm] \vektor{-a\cdot{}sin(t)\\b\cdot{}cos(t)}\qquad 0\le t\le 2\cdot{}\pi [/mm] $



Nun sagt meine Formelsammlung:

[mm] $s=\int [/mm] | [mm] \overrightarrow{\dot r} [/mm] | [mm] \;dt=\int \sqrt{\dot x^2 +\dot y^2}\;dt$ [/mm]

Also:

[mm] $s=\int_{0}^{2\pi} \sqrt{a^2*sin^2(t) +b^2*cos^2(t)}\;dt=4a*\int_{0}^{\pi/2} \sqrt{sin^2(t) +\bruch{b^2}{a^2}*cos^2(t)}\;dt$ [/mm]

[mm] $s=4a*\int_{0}^{2\pi} \sqrt{a^2*sin^2(t) +\bruch{b^2}{a^2}*(1-sin^2(t))}\;dt [/mm] = [mm] 4a*\int_{0}^{2\pi} \sqrt{sin^2(t) +\bruch{b^2}{a^2}-\bruch{b^2}{a^2}*sin^2(t)}\;dt$ [/mm]

$s= [mm] 4a*\int_{0}^{2\pi} \sqrt{(1-\bruch{b^2}{a^2})*sin^2(t)+\bruch{b^2}{a^2}}\;dt$ [/mm]

$s= [mm] 4a*\int_{0}^{2\pi} \sqrt{k^2*sin^2(t)+\bruch{b^2}{a^2}}\;dt$ [/mm]




Wenn ich nun von der Lösung ausgehe, dann bekomme ich:

$U= [mm] 4a*\int_{0}^{2\pi} \sqrt{1-k^2*sin^2(t)}\;dt$ [/mm]

$U= [mm] 4*\int_{0}^{2\pi} \sqrt{a^2-(a^2-b^2)*sin^2(t)}\;dt$ [/mm]

$U= [mm] 4*\int_{0}^{2\pi} \sqrt{a^2(1-sin^2(t))+b^2*sin^2(t)}\;dt$ [/mm]

$U= [mm] 4*\int_{0}^{2\pi} \sqrt{a^2*cos^2(t)+b^2*sin^2(t)}\;dt$ [/mm]


Und was hier unter der Wurzel steht ist ja:

[mm] $s=\int [/mm] | [mm] \overrightarrow{r} [/mm] | [mm] \;dt$ [/mm]


und nicht:

[mm] $s=\int [/mm] | [mm] \overrightarrow{\dot r} [/mm] | [mm] \;dt$ [/mm]


Wo habe ich mich verirrt?

Besten Dank für einen Hinweis.

LG, Martinius











Bezug
                        
Bezug
elliptisches Integral: Antwort
Status: (Antwort) fertig Status 
Datum: 19:19 Mi 25.08.2010
Autor: MathePower

Hallo Martinius,

> Hallo Al Chwarizmi,
>  
> besten Dank für die Antwort - dadurch bin ich der Lösung
> näher gekommen; ich habe aber noch folgendes Problem:
>  
> Die Parametergleichung für die Ellipse ist ja:
>  
> [mm]\vektor{x(t)\\y(t)}\ =\ \vektor{a\cdot{}cos(t)\\b\cdot{}sin(t)}\qquad 0\le t\le 2\cdot{}\pi[/mm]
>  
>
> Und ihre erste Ableitung:
>  
> [mm]\vektor{ \dot x(t)\\\dot y(t)}\ =\ \vektor{-a\cdot{}sin(t)\\b\cdot{}cos(t)}\qquad 0\le t\le 2\cdot{}\pi[/mm]
>  
>
>
> Nun sagt meine Formelsammlung:
>  
> [mm]s=\int | \overrightarrow{\dot r} | \;dt=\int \sqrt{\dot x^2 +\dot y^2}\;dt[/mm]
>  
> Also:
>  
> [mm]s=\int_{0}^{2\pi} \sqrt{a^2*sin^2(t) +b^2*cos^2(t)}\;dt=4a*\int_{0}^{\pi/2} \sqrt{sin^2(t) +\bruch{b^2}{a^2}*cos^2(t)}\;dt[/mm]
>  
> [mm]s=4a*\int_{0}^{2\pi} \sqrt{a^2*sin^2(t) +\bruch{b^2}{a^2}*(1-sin^2(t))}\;dt = 4a*\int_{0}^{2\pi} \sqrt{sin^2(t) +\bruch{b^2}{a^2}-\bruch{b^2}{a^2}*sin^2(t)}\;dt[/mm]
>  
> [mm]s= 4a*\int_{0}^{2\pi} \sqrt{(1-\bruch{b^2}{a^2})*sin^2(t)+\bruch{b^2}{a^2}}\;dt[/mm]
>  
> [mm]s= 4a*\int_{0}^{2\pi} \sqrt{k^2*sin^2(t)+\bruch{b^2}{a^2}}\;dt[/mm]
>  
>
>
>
> Wenn ich nun von der Lösung ausgehe, dann bekomme ich:
>  
> [mm]U= 4a*\int_{0}^{2\pi} \sqrt{1-k^2*sin^2(t)}\;dt[/mm]
>  
> [mm]U= 4*\int_{0}^{2\pi} \sqrt{a^2-(a^2-b^2)*sin^2(t)}\;dt[/mm]
>  
> [mm]U= 4*\int_{0}^{2\pi} \sqrt{a^2(1-sin^2(t))+b^2*sin^2(t)}\;dt[/mm]
>  
> [mm]U= 4*\int_{0}^{2\pi} \sqrt{a^2*cos^2(t)+b^2*sin^2(t)}\;dt[/mm]
>  
>
> Und was hier unter der Wurzel steht ist ja:
>
> [mm]s=\int | \overrightarrow{r} | \;dt[/mm]
>  
>
> und nicht:
>
> [mm]s=\int | \overrightarrow{\dot r} | \;dt[/mm]
>  
>
> Wo habe ich mich verirrt?


Bei Deinen Ausführungen  hast Du die falsche Parametrisierung gewählt.

Die hier benutzte Parametrisierung lautet:

[mm]\vektor{x(t)\\y(t)}\ =\ \vektor{a\cdot{}\blue{sin(t)}\\b\cdot{}\blue{cos(t)}}\qquad 0\le t\le 2\cdot{}\pi[/mm]

Dann stimmt das auch mit

[mm]s=\int | \overrightarrow{\dot r} | \;dt[/mm]


>  
> Besten Dank für einen Hinweis.
>  
> LG, Martinius



Gruss
MathePower

Bezug
                                
Bezug
elliptisches Integral: Dank
Status: (Mitteilung) Reaktion unnötig Status 
Datum: 15:45 Do 26.08.2010
Autor: Martinius

Hallo MathePower,

besten Dank für den Hinweis!

LG, Martinius

Bezug
Ansicht: [ geschachtelt ] | ^ Forum "Integralrechnung"  | ^^ Alle Foren  | ^ Forenbaum  | Materialien


^ Seitenanfang ^
www.vorkurse.de
[ Startseite | Mitglieder | Teams | Forum | Wissen | Kurse | Impressum ]